¿Cuál es la razón fundamental por la que no se puede romper la velocidad de la luz? ¿Por qué el universo quiere preservar la barrera superior a la velocidad de la luz tanto que ralentice el tiempo fácilmente en lugar de ver que se rompe la barrera de la velocidad?

Escuché esa pregunta de Richard Feynman una vez, en un pequeño grupo reunido con él después de una conferencia de profesores de física unos meses antes de su muerte. La pregunta fue hecha por un profesor de física jubilado de secundaria, y se hizo con mucho respeto. “Profesor Feynman, siempre quise saber por qué nada puede viajar más rápido que la velocidad de la luz. ¿Puedes explicar por qué? “Feynman tuvo ese brillo caprichoso en sus ojos y dijo con su extravagancia característica:” ¡No tenemos idea! Ah, claro, podemos hacer todo tipo de matemáticas y derivaciones elaboradas que, según afirmamos, ‘lo explican’. Pero el hecho es que solo tenemos un universo para estudiar. Y en nuestro universo, ningún objeto puede viajar más rápido que la velocidad de la luz “. (Lo pongo entre comillas, pero en verdad, no sé las palabras exactas, estoy tratando de dar la esencia de lo que dijo Después de todo, ¡fue hace treinta años y ahora soy probablemente mayor que el caballero ‘anciano’ que hizo la pregunta!)

La respuesta más fácil es porque algo tiene que ser constante para que nuestro sistema de matemáticas tenga sentido, y tener espacio o tiempo sean conflictos constantes con resultados experimentales que muestran tanto un universo en expansión como una variación en el tiempo percibido en función de la velocidad relativa.

La relatividad general de Einstein básicamente postula que la velocidad de la luz en el vacío es constante. Este postulado es más que una mera conjetura, porque la matemática detrás de él explica ciertos fenómenos observados y ha sido respaldado por una experimentación específica. Sin embargo, queda por demostrar, y probablemente nunca se demostrará definitivamente cierto (aunque es posible que sea definitivamente refutado), debido a las dificultades inherentes con respecto a la observación de la luz.

La razón de estas dificultades es que, estrictamente hablando, la luz es una forma pura de energía, y no podemos observar la energía directamente; solo podemos observar interacciones de energía con la materia (y la producción de energía a partir de la materia). La luz visible es visible porque las células en nuestros ojos reaccionan a la entrada de energía de los fotones en una banda particular de frecuencias, produciendo una señal electroquímica en nuestro sistema nervioso. De manera similar, la luz que no podemos ver todavía es detectable al interceptarla con otra materia sensible a ella; ciertos productos químicos son reactivos a los rayos UV, las antenas producen un voltaje y una corriente en presencia de ondas de radio, y la lista continúa. Sin embargo, no podemos simplemente extraer un fotón del espacio, examinarlo bajo un microscopio y luego dejarlo ir por su buen camino. En el famoso experimento de doble rendija, cualquier intento de observar las interacciones de los fotones entre las rendijas y la película reactiva (que normalmente produce varias líneas en la película debido a las interacciones de las ondas en lugar de las dos líneas esperadas en base a la comprensión de la luz como un partícula) hace que los efectos se descompongan, porque el equipo de observación interfiere con la interacción al capturar la luz antes de que alcance el objetivo original.

Sin embargo, es posible cronometrar las interacciones observadas. Podemos, por ejemplo, sincronizar dos relojes colocados a una corta distancia uno del otro, y luego conectarlos a un láser u otro equipo emisor de EMR para producir pulsos de luz, y aumentar la distancia de un aparato en relación con el otro , observando la diferencia horaria entre los pulsos producidos por cada reloj. ∆d / ∆t = v, que en este caso es c (o extremadamente cerca de él; siempre hay una latencia adicional inherente a la transmisión de energía a través de un medio más denso que un vacío, incluido nuestro equipo de medición).

Sin embargo, cuando los científicos intentaron aumentar el tiempo de demora para aumentar la precisión de la medición al aumentar aún más la distancia, descubrieron que cuando los relojes volvían a su proximidad original, no siempre volvían perfectamente sincronizados, aunque Los relojes en sí, basados ​​en la descomposición del cesio, eran muy precisos y no se desincronizaban cuando estaban sentados uno al lado del otro. Ese fenómeno también es predicho por la relatividad general; Debido a que la velocidad de la luz es una constante universal, que no varía según la velocidad del emisor como puede ser una partícula masiva, lo que cambia es la percepción del tiempo entre dos marcos de referencia en movimiento dentro del espacio-tiempo.

Como experimento mental, considere dos objetos, que se mueven entre sí a cierta velocidad v = v1 + v2, donde v1 y v2 son las velocidades de cada objeto en relación con un único punto fijo en el espacio. Uno de ellos tiene un emisor de fotones de algún tipo, que emite un fotón a intervalos regulares, y estos fotones viajan al otro objeto que tiene un colector. En el tiempo t1 , cuando los dos objetos están separados por una distancia, el emisor envía un fotón. En el momento t2 = t1 + δt , en cuyo punto los objetos están separados d + vδt , se envía un segundo fotón. Estos fotones viajan en c hacia el receptor . ¿Cuánto tardan? Bueno, el primer fotón tiene que viajar d / c segundos para llegar a donde estaba el objeto receptor. Durante ese tiempo, el receptor se ha alejado v2d / c más lejos de ese punto, por lo que la luz también tiene que recorrer esa distancia. Durante ese tiempo, el objeto se movió aún más, pero finalmente, la luz llega al receptor en un momento t3 tal que ct3 = v2t3 + d. El segundo fotón se emitió cuando los objetos estaban más separados, de modo que el fotón tiene que viajar una distancia más larga para llegar al receptor; eso lleva tiempo t4 tal que ct4 = v2t4 + d + vδt. Encontramos de esta matemática que t2-t1 = δt para cualquier v > 0; entonces, el receptor observa una desaceleración del paso del tiempo en el marco de referencia del emisor, debido a su velocidad relativa.

Por esta misma matemática, dos objetos en el espacio-tiempo pueden moverse uno con respecto al otro en una v> c , y aún ser observables entre sí; Como ambos se mueven a v relación con un punto fijo en el espacio, los fotones aún pueden “alcanzar” al receptor porque no se ven afectados por la velocidad relativa del emisor. Sin embargo, la dilatación del tiempo sería muy grande.

Aquí es donde las cosas se ponen interesantes: supongamos que v1 = 0 en relación con un punto fijo en el espacio, y entonces v = v2. Supongamos además que v = c ; El receptor se mueve a la velocidad de la luz lejos del emisor. ¿Lo que pasa? Bueno, en cualquier d> 0 , el protón enviado por el emisor nunca alcanzará al receptor. Desde el marco de referencia del receptor, el emisor no puede observarse a tiempo; Mientras se mantenga esta velocidad, no se observa el paso del tiempo del emisor. Desde una definición muy estricta de v = d / t , debido a la expansión del espacio-tiempo, creemos que no podemos ver todo el universo, no porque esos objetos se muevan a través del espacio-tiempo en o más allá de c , sino porque hay más espacio, y, por lo tanto, se agrega más distancia en una unidad de tiempo entre nosotros y los objetos debido a la expansión de lo que la luz puede viajar al mismo tiempo.

Consideremos otro ejemplo; v1 = -c y v2 = 0. Ahora, el emisor viaja hacia el receptor a la velocidad de la luz, cubriendo cierta distancia arbitraria d . Desde el momento en que el emisor alcanza c , cualquier fotón emitido por él viajará tan rápido como el objeto mismo. Hasta que recibamos un fotón del emisor, no sabremos que está allí; entonces, una vez que llega, tal vez disminuyendo a v parecerá, en un instante, estar en todas partes a lo largo de su camino de viaje a la vez.

Finalmente, consideremos v1> c . El emisor ahora se mueve más rápido a través del espacio que los fotones que está emitiendo. Suponiendo que eso podría suceder (hay muchas paradojas interesantes sobre lo que le sucedería al fotón; el emisor lo “colisionaría” efectivamente en el mismo instante en que se emite, entonces, ¿sería recapturado?), El objeto se llegar al receptor más rápido que su luz observable. En el momento en que disminuyó a v , observaríamos que el objeto está en un punto particular en el espacio. Sin embargo, como los fotones emitidos mientras el objeto viajaba más rápido de lo que la luz nos alcanzaba, el objeto sería observable en dos lugares a la vez; en su posición actual donde está emitiendo fotones en reposo relativo, y en puntos progresivamente más alejados de su posición actual; efectivamente parecería estar viajando en reversa al mismo tiempo que descansaba.

Todas estas cosas también suponen que los dos objetos son más o menos fuentes puntuales; Partículas individuales que toman efectivamente un solo punto en el espacio y se mueven en relación con solo una partícula más. Dos o más partículas, incluso a nivel subatómico, que se mueven juntas a través del espacio-tiempo más rápido que la luz, en términos de las interacciones que tienen entre sí, serían un “emisor” y un “observador” entre sí, y porque serían moviéndose a través del espacio-tiempo más rápido que la información que tienen que intercambiar para existir como tal, no pueden existir como tal.

Esta es, en última instancia, la razón por la que consideramos que c es un límite de velocidad universal. Nada puede moverse más rápido que la luz, porque las interacciones y fuerzas que definen la materia ocurren a la velocidad de la luz (podemos respaldar esto con la experimentación en enredos cuánticos y otras interacciones), y por lo tanto, la materia no puede existir si viaja a través del espacio más rápido que la luz. Por la misma razón, solo la luz puede moverse a la velocidad de la luz, porque en el punto donde una partícula masiva alcanza c, las interacciones que la mantienen unida y realmente la definen, que se mueven en c , no pueden ocurrir, y prácticamente todo se aceleró a c efectivamente se convierte en luz.

No soy físico, así que perdóname si me equivoco, aquí. Y, los físicos (o personas que entienden la física mejor que yo), corrijan mis errores. Gracias. Principalmente estoy respondiendo esto para ver si puedo. Aprendo mejor tratando de responder preguntas.

Cuanto más pesado es algo, más difícil es moverse. Eso es sentido común, y lo experimentamos todos los días. Esperamos que sea mucho más fácil empujar un carro de juguete que un carro real o una canica que una bala de cañón. Cuanto más pesa algo, más energía necesitamos para impulsarlo.

(Por razones de precisión, debería dejar de hablar sobre el peso y la “pesadez” y hablar, en cambio, sobre la masa. Cuanto más masa tiene algo, más energía se necesita para moverlo. Pero dado que el peso, que realmente solo existe cuando la masa se encuentra con la gravedad —Es intuitivo, lo seguiré evocando. Piense en ello como una metáfora).

Entonces, digamos que lanzamos una nave espacial realmente grande, digamos el tamaño de Starship Enterprise. Es de esperar que requiera mucha energía como propulsión. Peor aún, cuanto más rápido van las cosas, más pesan (más masa tienen), por lo que si seguimos tratando de hacer que el Enterprise se mueva más y más rápido, necesitaremos más y más energía para impulsarlo. A cierta velocidad, necesitaremos tanta energía para impulsarlo, usaremos toda la energía del Universo. Llamemos a esa velocidad S.

Como recapitulación, S es la velocidad que la Enterprise puede viajar si usamos toda la energía del Universo para impulsarla.

¿Qué pasa si queremos que se mueva más rápido que S? Bueno, estamos sin energía, por lo que tenemos que hacer que el barco sea más ligero (tenemos que hacer que tenga menos masa). Si desechamos, digamos, a todos los miembros de la tripulación, el barco no pesará tanto. Lo que significa que obtendremos más velocidad de toda la energía en el Universo. Podremos mover el barco a la velocidad S [1], que es una velocidad que es más rápida que S.

Si desechamos todas las sillas y mesas, podremos mover la nave en S [2]. Si desechamos todas las naves lanzaderas, podremos movernos en S [4]. Etc.

Sigamos rompiendo partes de la empresa para que se mueva cada vez más rápido. Separemos tanto, que solo le queda una unidad de peso (realmente masa). No estoy seguro de qué es el barco ahora, pero imagina que es una pequeña especificación que es tan ligera como un objeto puede ser. Se mueve a, digamos, S [5,000].

¿Cómo podríamos hacer que se mueva aún más rápido que S [5,000]? ¡Al hacer que no pese nada en absoluto! Si cuanto más pesa algo, más energía necesita para empujarlo, la forma de obtener el mayor rendimiento de su inversión es hacer que el objeto que empuja no pese nada. ¡Imagínese lo rápido que podría lanzar una pelota de béisbol si no tuviera peso alguno!

Por desgracia, no podemos hacer que el Enterprise no pese nada en absoluto, pero hay algo que no pesa nada: un fotón. (Nuevamente, estrictamente hablando, un fotón no tiene masa). Un fotón es una partícula de luz. Como las partículas de luz no pesan nada, pueden viajar más rápido, es posible que cualquier cosa viaje, es decir, en el sistema que inventé, tal vez S [5,001].

Resulta que sabemos el valor real de S [5,001]. Es alrededor de 180 mil millas por segundo. Así de rápido puede viajar algo si no pesa nada. Y, si piensas en lo que escribí arriba, si no pesa nada, no requiere energía para empujarlo. Tan pronto como agregue un poco de peso a algo, requiere energía para empujar, y cuanto más rápido lo empuje, más energía requiere. Para empujar un objeto con peso (masa) a 180 mil millas por segundo, necesitarías más energía de la que existe en el Universo. ¡Así que no tiene suerte, Capitán Kirk!

La pregunta persistente aquí es ¿por qué 180 mil millas por segundo? ¿Qué tiene de mágico eso? ¿Por qué un fotón no puede viajar más rápido que eso? (¿Por qué la luz no puede viajar más rápido que la velocidad de la luz?)

Hasta donde yo sé (nuevamente, físicos, corríjanme si me equivoco), la respuesta es porque . Porque así es como se construye nuestro Universo. Es una ley física que adquirimos al ser producto del Big Bang. Si hay otros universos, quizás la constante sea más rápida (o más lenta) en algunos de ellos.

Cuando llegamos a un cierto nivel base de “¿por qué?” preguntas, la respuesta siempre será “porque”. Nuestro universo parece ser un juego con ciertas reglas (las leyes de la física). ¿Por qué esas reglas y no otras reglas?

Porque…

ACTUALIZACIÓN: le pedí a Joshua Engel (que tiene un título en física) que revise esto y lo critique. Esto es lo que me envió en un correo electrónico, publicado con su permiso:

Es bastante bueno. No es realmente “fundamental”, pero la mayoría de los que preguntan no están realmente preparados para respuestas “fundamentales”. Todo lo que podemos hacer es lanzar una serie de aproximaciones y esperar que una de ellas se ajuste al nivel real en el que se encuentran.

Podemos decir un poco más que “porque” cuando se trata de por qué la velocidad de la luz es la que es, aunque rápidamente sale de un área de la que me siento cómodo hablando. Dentro de la relatividad especial, se puede decir que es porque el espacio “simplemente es” la métrica de Minkowski (s ^ 2 = x ^ 2 + y ^ 2 + z ^ 2- (ct) ^ 2), y es el hecho de que s es independiente de todos los observadores que le permiten resolver c y obtener la respuesta que obtenemos. (En particular, s = 0 para la luz).

Si uno entiende la relatividad general, puede llevarlo unos pasos más allá, a constantes fundamentales de la forma del universo, como el tensor Ricci y el escalar Ricci, y generalizar eso a las branas y ese galimatías como ese, pero eso está mucho más allá mi nota salarial

Así que no importa a dónde vaya, siempre hay un “simplemente es”, pero para prácticamente todas las personas el “solo es” tiene más que ver con la cantidad de estudios que han realizado en lugar de lo que se sabe o se puede saber .

En cambio, lanzamos una variedad de argumentos, todos basados ​​en lo mismo que los usuarios, y vemos qué es lo que “se pega”.

La única objeción: los fotones tienen masa, en el sentido de que tienen energía y E = m (con una constante adicional de c ^ 2 para que las unidades funcionen). No tienen ninguna energía si se detuvieran, porque no tienen masa en reposo . Obtienen su energía del componente E = pc adicional que la mayoría de las personas deja fuera de la ecuación: no tienen masa pero sí tienen impulso. Si te golpea un fotón, te empujan; así funcionan las velas solares.

Ahí es exactamente donde va su argumento: cuando lleva el límite de masa a cero, obtiene un fotón, que no tiene masa en reposo y, por lo tanto, nunca puede estar en reposo, o de hecho a cualquier velocidad que no sea c, que es el único uno donde los diversos infinitos se cancelan para producir un valor no infinito.

tl; dr: si nuestro universo es causal y local, deberíamos esperar una velocidad máxima a la cual los eventos pueden causar otros eventos.

Juguemos a Dios e imaginemos que estamos diseñando un nuevo universo. El universo es un espacio, ocupado por cosas, y las cosas suceden en él, que son muy, muy vagas. ¿Qué principios rectores fundamentales deberíamos usar para construir nuestro nuevo universo?

Si no desea que todo sea totalmente caótico, un principio rector que podría tener es la causalidad : cuando suceden dos cosas A y B en nuestro universo, podrían no estar relacionadas o relacionadas por causalidad: A causó B o B causó A [1], y las leyes de física que vas a diseñar te dirían cómo ocurre esta causalidad.

Ahora, imagine dos observadores moviéndose a través de su nuevo universo, observando los eventos A y B. Suponga que el observador 1 determina que A causó B : ¿debería el observador 2 estar de acuerdo con esta observación? Podrías decir ‘no’, por supuesto, e inventar algunas reglas para hacer que esto funcione [2], pero la alternativa es más limpia, y también verdadera para el universo real: todos los observadores están de acuerdo en que A causó B , sin importar qué. Esto le da un orden de los eventos en el universo, que en el universo real se mide por el tiempo.

Si tenemos causalidad en nuestro nuevo universo, la siguiente pregunta natural es preguntar cómo los eventos se vuelven causalmente conectados. ¿Puede un evento que ocurre en cualquier punto del universo, afectar a cualquier otro punto? ¿Puede el tiempo entre dos eventos que están causalmente conectados ser arbitrariamente cortos? Una vez más, podría decir ‘sí’, pero esto sería un poco inquietante: algo que está sucediendo en este momento en la galaxia de Andrómeda, por ejemplo, podría matarnos al instante.

Si, por el contrario, dice ‘no’, entonces ha introducido otro principio rector en el diseño del nuevo universo: la localidad . Algo que sucede en algún momento en el espacio y el tiempo afecta su entorno inmediato en el espacio y el tiempo solamente, y no afecta de inmediato a todo el universo de una vez.

Dada la causalidad y la localidad, quizás la siguiente pregunta más natural es preguntar qué tan local es ‘local’, si A es un evento y B es otro lugar, podemos imaginar una “esfera de causalidad” y preguntar qué tan pronto puede A causar algo en B. Esto es arbitrario, y si lo configura al infinito, vuelve al universo no local en el párrafo anterior, que como dijimos es un poco incómodo y no es consistente con el universo real. Por lo tanto, debe establecer algún tipo de velocidad máxima a la que un evento puede causar otro evento. En nuestro universo, esta es precisamente la velocidad de la luz.

Veamos las consecuencias de tal elección: primero, ¿deberían todos los observadores estar de acuerdo con esta velocidad máxima? Bueno, sí, de lo contrario no habría un máximo. Así que ahí lo tienes: si quieres un universo local causal, debes esperar una velocidad máxima a la que los eventos puedan causar otros eventos. ¿Tiene que ser la velocidad de la luz? Supongo que no: pero integrado en las ecuaciones del electromagnetismo, como lo verifican muchos, muchos experimentos, es una velocidad en la que todos los observadores parecen estar de acuerdo, que es precisamente la velocidad a la que se propaga la luz. Eso debería darte una muy, muy buena pista de que esta es la misma velocidad máxima que estás buscando en nuestro universo (¡por supuesto, los físicos se desviaron un poco antes de darse cuenta de esto!).

En resumen, si desea un universo en el que pueda conectar eventos causalmente, y un universo en el que las causas solo provoquen efectos locales, entonces deberíamos esperar algún tipo de límite de velocidad en el que los efectos de una causa puedan propagarse. Este máximo debe ser independiente del observador (de lo contrario, no sería un máximo). La velocidad de la luz es precisamente una velocidad independiente del observador, por lo que esperamos que sea precisamente ese límite de velocidad. Cada ecuación en la física moderna tiene estos supuestos en su núcleo, y hasta ahora todo parece estar funcionando relativamente bien, ¡así que estamos bastante seguros de que es correcto!

[1] Se podría imaginar, supongo, un universo en el que cada evento no está relacionado con cada evento, pero ese sería un universo en el que los humanos nunca serían capaces de darle sentido a nada, y ese es casi seguro que no sea el caso en el universo real

[2] Una manera fácil es decir que hay una forma especial de observar el universo en el que toda la causalidad se determina correctamente.

Siempre he tenido esta pregunta y leer algunas de las respuestas aquí ayuda un poco. Pero la mejor explicación laica sobre la velocidad de la luz que he leído es del hilo de reddit: ELI5 ¿Por qué viaja la luz? • / r / explicar como imitivo. Solo lo citaré aquí:

Todo, por naturaleza de simplemente existente, se está “moviendo” a la velocidad de la luz (que realmente no tiene nada que ver con la luz: más sobre eso más adelante). Sí, eso te incluye a ti.

Nuestra comprensión del universo es que la forma en que percibimos el espacio y el tiempo como cosas separadas es, para ser sincero, incorrecto. No están separados: el universo está hecho de “espacio-tiempo”, todo en una palabra. Un año y un año luz describen diferentes cosas en nuestra vida cotidiana, pero desde el punto de vista de un físico, en realidad son exactamente lo mismo (dependiendo de qué tipo de física estés haciendo).

En nuestra vida cotidiana, definimos el movimiento como la distancia recorrida durante una cierta cantidad de tiempo. Sin embargo, si las distancias y los intervalos de tiempo son exactamente lo mismo , eso de repente se vuelve completamente sin sentido. “¡Viajé un pie por cada pie que viajé” es una declaración absolutamente absurda!
La forma en que funciona es que todo en el universo viaja a través del espacio-tiempo a una velocidad que llamaré “c” en aras de la brevedad.

Recuerde, el movimiento en el espacio-tiempo no tiene sentido, por lo que tiene sentido que nada pueda ser “más rápido” o “más lento” a través del espacio-tiempo que cualquier otra cosa. Todos y todo viaja a un pie por pie, así es … cómo funciona.

Obviamente, sin embargo, las cosas parecen tener velocidades diferentes. La razón que sucede es que el tiempo y el espacio son ortogonales , lo cual es una especie de término elegante para “en ángulo recto entre sí”. El norte y el este, por ejemplo, son ortogonales: puede viajar tan lejos como desee directamente hacia el norte, pero no afectará en absoluto a dónde se encuentra en términos de este / oeste.

Al igual que puede viajar hacia el norte sin viajar hacia el este, puede viajar a través del tiempo sin que afecte dónde se encuentra en el espacio. Por el contrario, puede viajar a través del espacio sin que afecte dónde se encuentra en el tiempo.
Estás (presumiblemente) sentado en tu silla ahora mismo, lo que significa que no estás viajando por el espacio en absoluto. Sin embargo, dado que tiene que viajar a través del espacio-tiempo a c (velocidad de la luz), eso significa que todo su movimiento es a través del tiempo.

Por cierto, esta es la razón por la que ocurre la dilatación del tiempo: algo que se mueve muy rápido en relación con usted se mueve a través del espacio, pero dado que solo pueden viajar a través del espacio-tiempo en c, tienen que moverse más lentamente a través del tiempo para compensar (desde su punto de vista).

La luz, por otro lado, no viaja en el tiempo en absoluto. La razón por la que no es algo complicada, pero tiene que ver con el hecho de que no tiene masa.

Algo que no se mueve que tiene masa puede tener energía: eso es lo que significa E = mc2. La luz no tiene masa, pero tiene energía. Si conectamos la masa de luz a E = mc2, obtenemos 0, lo que no tiene sentido porque la luz tiene energía. Por lo tanto, la luz nunca puede ser estacionaria.

No solo eso, sino que la luz nunca puede ser estacionaria desde la perspectiva de nadie . Como, como todo lo demás, viaja en c a través del espacio-tiempo, eso significa que toda su “velocidad del espacio-tiempo” debe ser a través del espacio, y nada de eso es a través del tiempo.

Entonces, la luz viaja en c. No por casualidad, a menudo escuchará c referida como la “velocidad de la luz en el vacío”. Realmente, sin embargo, es la velocidad a la que todo viaja, y resulta ser la velocidad a la que viaja la luz a través del espacio porque no tiene masa.

Por cierto, esto también cubre la pregunta común de ELI5 de por qué nada puede viajar más rápido que la luz y por qué las cosas con masa no pueden viajar a la velocidad de la luz. Como todo se mueve a través del espacio-tiempo en c, nada puede excederlo (y no, viajar hacia atrás en el tiempo no solucionaría eso). Además, las cosas con masa siempre pueden ser “estacionarias” desde la perspectiva de alguien (como la suya), por lo que siempre tienen que moverse a través del tiempo al menos un poco , lo que significa que nunca pueden viajar por el espacio tan rápido como lo hace la luz. Tendrían que viajar a través del espacio-tiempo más rápido que c para hacer eso, lo cual, nuevamente, no es posible.

La relación entre El tiempo y el espacio se explican maravillosamente aquí. Y me da una comprensión general bastante buena sobre la relatividad.

Corté la respuesta a un lenguaje muy simple ya que todas las otras respuestas son muy complejas y subjetivas.

La RELATIVIDAD lo explicará . Todos sabemos sobre la dilatación del tiempo. A velocidades cercanas a la luz, el tiempo se mueve más lento, entonces ¿Qué sucede a la velocidad de la luz? El tiempo se detiene y el momento en que alcanzas la velocidad de la luz se vuelve estacionario para el mundo exterior, para ti no existe el concepto del tiempo.

Además, velocidad = cambio en la distancia / cambio en el tiempo. El cambio en el tiempo es 0 y el cambio en la distancia en algún valor finito. Significa que la velocidad se volvió infinita en el momento en que alcanzaste la velocidad de la luz. Pero, esta es una contradicción de que estás viajando con velocidad infinita. Significa que solo puede acercarse a esa velocidad a medida que nos acercamos al infinito, por eso siempre se dice que 99.99999% de velocidad de la luz.

Desde la perspectiva de un fotón

Uno de los métodos que Einstein usó para ayudar a formular su teoría de la relatividad especial fue visualizar cómo sería el universo desde la perspectiva de un fotón. Einstein vio que la vida como un fotón sería bastante extraña. Por ejemplo, si fueras un fotón, el tiempo no tendría sentido para ti. Todo parecería suceder instantáneamente.
Imagine por un momento que usted es un pequeño fotón feliz creado por una estrella en otra galaxia a unos 4 mil millones de años luz de distancia. Desde mi perspectiva aquí en la Tierra, te tomó exactamente 4 mil millones de años viajar desde esa estrella hasta que llegaste a mi retina. Desde tu perspectiva, en un instante fuiste creado y luego al siguiente, estás rebotando en mi globo ocular. No experimentaste el paso del tiempo. Tu nacimiento y muerte ocurrieron instantáneamente.
Esto se debe a que el tiempo se ralentiza a medida que se acerca a la velocidad de la luz y, cuando se detiene, se detiene por completo. Esta es también otra razón por la cual nada puede ir más rápido que la luz. Sería como reducir la velocidad de un automóvil hasta detenerse, y luego intentar ir más despacio que detenerse por completo.
Uno debería pensar en la velocidad de la luz como ‘velocidad infinita’. Un error común es pensar que la velocidad de la luz es como cualquier otra velocidad finita. La velocidad de la luz es solo finita desde la perspectiva del observador externo; desde la perspectiva de un fotón, es infinito. Si te mueves exactamente a la velocidad de la luz, podrías ir a cualquier parte, sin importar qué tan lejos, en exactamente cero segundos.

Todo está en los supuestos de la Relatividad Especial (SR).

SR tiene solo 1 suposición: las leyes de Física son las mismas en todos los marcos de referencia.
¡Veamos cómo esta suposición nos da todos estos resultados alucinantes!


SR surgió como consecuencia de las ecuaciones de Maxwell del electromagnetismo. De las 4 ecuaciones de Maxwell, obtenemos la ecuación de onda:

[matemáticas] \ nabla ^ 2 E = \ frac {1} {c ^ 2} \ frac {\ partial ^ 2} {(\ partial t) ^ 2} E [/ matemáticas]

¿Ves ese término [matemático] 1 / c ^ 2 [/ matemático] que yace ahí? ¡Eso nos dice que la velocidad de vacío de la luz es c pero no menciona qué marco de referencia se debe usar! Esto condujo a todo tipo de teorías, como la teoría del éter luminífero, etc., en un intento por encontrar el marco en el que la velocidad de la luz es c .

Sin embargo, cada una de estas teorías demostró estar equivocada por experimentos. Uno de los experimentos más fantásticos para hacer esto fue el experimento de Michelson-Morley, que rechazó por completo la idea de un éter.

Fue el genio de Einstein el que se dio cuenta de que esta velocidad de vacío de la luz es invariable en todos los marcos de referencia, es decir, si consideramos que la suposición de SR es verdadera, automáticamente obtenemos la misma velocidad de la luz en todos los marcos de referencia y esa velocidad es c . Entonces, no importa cuán rápido se mueva, la velocidad de la luz para ese observador será c.


¿Y qué, preguntas?

Este único resultado lleva a todos los resultados de SR. Obtenemos las transformaciones de Lorentz a partir de este resultado, y también obtenemos la famosa ecuación

[matemáticas] E ^ 2 = (mc ^ 2) ^ 2 + (pc) ^ 2 [/ matemáticas]

de este resultado

Todo esto será útil cuando respondamos su pregunta.


Verá, la teoría de la relatividad tiene que ver con lo que ve un observador en relación con otro observador. Por lo tanto, necesitamos tener al menos dos observadores para que cualquiera de los resultados de SR tenga sentido.

Por ejemplo, si fuera solo usted, entonces, en su propio marco, siempre está en reposo, y todos sus relojes y longitudes son normales.

Ahora, suponga que hay una nave espacial que se mueve con respecto a usted con cierta velocidad v . Luego, obtenemos de las ecuaciones de SR:

[matemáticas] t = \ gamma \ tau [/ matemáticas]

dónde
[matemáticas] \ gamma = \ frac {1} {\ sqrt {1 – \ frac {v ^ 2} {c ^ 2}}} [/ matemáticas]
[matemáticas] \ tau [/ matemáticas] es el tiempo medido en el marco de la nave espacial
[matemáticas] t [/ matemáticas] es el tiempo medido en su marco.

Hay un par de sutilezas aquí. Primero, para las personas en la nave espacial, el tiempo todavía fluye normalmente, es decir, no sienten que el tiempo va más lento, ni nada de eso.

En segundo lugar, para usted en la Tierra, vemos que a medida que la velocidad de la nave espacial en su marco v se acerca a la velocidad de la luz, el tiempo en la nave será mucho más lento, porque [matemáticas] \ gamma [/ matemáticas] es mucho más grande. Por lo tanto, a medida que la nave espacial se acerca cada vez más a c, el tiempo en la nave parece ser más y más lento para usted en la tierra.

Otra consecuencia inmediata de SR es que a medida que te acercas a c, la energía requerida para acelerar te hace más y más alta, y para llegar a c requiere una cantidad infinita de energía. Por eso, solo las partículas sin masa viajan a la velocidad de la luz.


Entonces, usted ve que la única suposición de SR (que considero bastante razonable), las leyes de la física deben permanecer igual en todos los marcos de referencia, explica todo lo extraño que está sucediendo.

Espero que esto aclare parte de la confusión con respecto a la desaceleración del tiempo y otras cosas por el estilo. ¡Recuerde, todo es relativo ! SR solo tiene sentido cuando hay dos observadores que miran el mismo evento y, por lo tanto, tienen observaciones diferentes. Para un solo observador, no hay dilatación del tiempo ni contracción de la longitud ni ninguno de los efectos extraños, ¡porque siempre están en reposo en su marco de referencia!

¡Feliz viaje!

Supongamos que tienes muchos juegos de bolas; cada conjunto tiene 15 bolas tan dispuestas que la altura del conjunto es 2.3. Si combina dos juegos de bolas, la altura total no se convierte en 4.6; se convierte en 3.2. Si combina tres juegos de bolas, la altura total no se convierte en 6.9; se convierte en 4.1.

Suponga que está combinando estos juegos de bolas estirando las manos y puede estirar las manos hasta una altura máxima de 9. Luego, la altura máxima posible para un juego combinado de bolas será de 9.

Por lo tanto, en este caso, la altura no se suma linealmente; Se agrega con un poco de disminución y límites a un cierto límite. Puede decir que la altura en este caso no es un escalar regular ; Es un escalar cónico con tapa .

Del mismo modo, la velocidad no se suma linealmente; Se agrega con un poco de disminución y límites a un cierto límite. Puedes decir que la velocidad no es un vector regular ; Es un vector con punta cónica .

Si agrega velocidad [matemática] v_2 [/ matemática] a la velocidad [matemática] v_1 [/ matemática], no se convierte en [matemática] v_1 + v_2 [/ matemática]; se convierte en [matemáticas] (v_1 + v_2) / {(1 + || v_1 ||. || v_2 || / c ^ 2)} [/ matemáticas].

De acuerdo con esta ecuación, la velocidad combinada se estrecha y se limita; es menor que [math] v_1 + v_2 [/ math]; no puede exceder [math] c [/ math].

Editar:

  • No he respondido la pregunta. Prefiero tratar de llegar a una razón subyacente. Todavía estoy en una fase de investigación. (¿Quien no lo es?)
  • La disminución de la altura en la ilustración se debe a la gravedad del entorno. La disminución gradual de la velocidad puede deberse a la interacción masa-momento.
  • El límite de altura en la ilustración se debe a la capacidad del agente. La limitación de velocidad puede deberse a cómo se recupera la información del espacio-tiempo.
  • Aquí: 2.3 significa 23 dividido por 10; [matemática] {v_1}. {v_2} [/ matemática] significa producto punto de vectores.
  • He introducido aquí conceptos de reducción y limitación de escalares y vectores. Estos conceptos también pueden ser útiles en otras áreas.

Como varios otros comentaristas ya señalaron, si su pregunta se interpreta literalmente como se le preguntó, entonces la respuesta es que, de acuerdo con la relatividad especial, todo viaja exactamente a la velocidad de la luz en el espacio-tiempo. Para darle lo que podría encontrar una respuesta más satisfactoria que algunas de las que ya se dieron, vale la pena explicar primero lo que realmente significa la afirmación de que “todo viaja a la velocidad de la luz en el espacio-tiempo”, y luego, usar las ideas obtenidas de esto para abordar la pregunta que está haciendo.
Mi respuesta involucrará algunas matemáticas, y dado que no conozco tus conocimientos matemáticos, lo mantendré muy fácil y te guiaré a mano. Confía en mí, no será muy difícil, y si haces el esfuerzo de seguir la cadena de razonamiento que se presenta a continuación, puedes descubrir algunos aspectos de la relatividad que pueden sorprenderte, y tal vez incluso una respuesta satisfactoria que responda directamente a la pregunta que haces. están preguntando.
Así que aquí va: la fórmula relevante para esto es

[matemáticas] \ beta ^ 2 + \ frac {1} {\ gamma ^ 2} = 1 [/ matemáticas]

Aquí [matemáticas] \ beta = \ frac {v} {c} [/ matemáticas] es la relación entre la velocidad del objeto observado y la velocidad de la luz y

[matemáticas] \ frac {1} {\ gamma} = \ sqrt {1- \ frac {v ^ 2} {c ^ 2}} [/ matemáticas]

es el inverso del conocido factor de Lorentz [math] \ gamma [/ math].
Sin embargo, a menos que tenga un fondo de física o matemática relativamente fuerte, esto probablemente no parezca muy obvio. Así que tratemos de entender esta relación más profundamente.
El primer paso es darse cuenta de que la inversa del factor de Lorentz puede considerarse como un parámetro de la rapidez con que se observa que un objeto “envejece”. Para ver esto, primero considere la analogía de cómo el movimiento rectilíneo uniforme en el espacio [matemática] v [/ matemática] (es decir, el movimiento a lo largo de una línea recta) multiplicado por el tiempo da la distancia recorrida [matemática] d [/ matemática]:

[matemáticas] d = vt [/ matemáticas]

Ahora mostraré que puedes pensar en el factor inverso de Lorentz de una manera completamente análoga, donde en lugar de la distancia [matemática] d [/ matemática] es el tiempo que se “atraviesa”.
Primero, considere que en la relatividad especial hay dos parámetros de tiempo, el tiempo apropiado [matemática] \ tau [/ matemática] y el tiempo coordinado [matemática] t [/ matemática]. El tiempo apropiado es el tiempo que pasa en el marco de descanso de un objeto observado, y el tiempo de coordinación es la cantidad de tiempo que usted, el observador, toma para ser equivalente a la cantidad de tiempo apropiado que observó que pasó en el marco de descanso del objeto observado Los dos están relacionados entre sí por la siguiente ecuación:

[matemáticas] \ tau = \ frac {t} {\ gamma} [/ matemáticas]

Para tener alguna intuición para esto, primero considere una situación en la que observe un objeto que esté en reposo con respecto a usted. Eso significa que

[matemáticas] v = 0 [/ matemáticas]

y por lo tanto

[matemáticas] \ frac {1} {\ gamma} = \ sqrt {1- \ frac {v ^ 2} {c ^ 2}} = 1 [/ matemáticas]

y por lo tanto

[matemáticas] \ tau = t [/ matemáticas]

Por lo tanto, su tiempo de coordenadas y el tiempo adecuado del objeto son los mismos porque están en reposo uno con respecto al otro. Pero ahora suponga que está observando algo que se mueve con respecto a usted. Luego

[matemáticas] v> 0 [/ matemáticas]

y por lo tanto

[matemáticas] \ frac {1} {\ gamma} = \ sqrt {1- \ frac {v ^ 2} {c ^ 2}} <1 [/ matemáticas]

y por lo tanto

[matemáticas] \ tau

y cuanto más grande [matemática] v [/ matemática], menor [matemática] \ frac {1} {\ gamma} = \ sqrt {1- \ frac {v ^ 2} {c ^ 2}} [/ matemática] y el más pequeño [math] \ tau [/ math] en relación con [math] t [/ math].

Hasta ahora, he conceptualizado la situación como se hace en cualquier libro de física de primer año. Pero ahora aprovechemos lo que acabamos de encontrar multiplicándolo por la velocidad de la luz y definiendo una nueva cantidad que, en esencia, nos dice la “velocidad de envejecimiento” en las mismas unidades dimensionales que la velocidad (en el espacio):

[matemáticas] \ frac {c} {\ gamma} \ equiv v _ {\ tau} [/ matemáticas]

Aquí el símbolo [math] \ equiv [/ math] significa “se define como” y [math] v _ {\ tau} [/ math] es una cantidad que podría llamarse “movimiento en el momento adecuado”. Ahora volvamos a la ecuación que relaciona el tiempo apropiado para coordinar el tiempo [matemáticas] \ tau = \ frac {t} {\ gamma} [/ matemáticas] y multiplique ambos lados por la velocidad de la luz. Entonces tenemos

[matemáticas] c \ tau = c \ frac {t} {\ gamma} = v_ \ tau t [/ matemáticas]

Compare esto con la ecuación anterior para el movimiento en el espacio, [matemática] d = vt [/ matemática] y verá que las dos son exactamente análogas: evidentemente, la cantidad [matemática] c \ tau [/ matemática] es análoga a una distancia , de hecho, no es difícil mostrar (pero no es directamente relevante a su pregunta) que esta “distancia” es en realidad un análogo de cuatro dimensiones de distancia tridimensional en el espacio llamado “intervalo espacio-tiempo” (es diferente de un distancia verdadera en que cuando está al cuadrado, se permiten valores negativos). Consideremos ahora la relación con la que comenzamos, recordemos que fue

[matemáticas] \ beta ^ 2 + \ frac {1} {\ gamma ^ 2} = 1 [/ matemáticas]

Multiplica ambos lados por la velocidad de la luz al cuadrado [matemática] c ^ 2 [/ matemática] y obtendrás

[matemáticas] c ^ 2 \ beta ^ 2 + \ frac {c ^ 2} {\ gamma ^ 2} = c ^ 2 [/ matemáticas]

o, usando [math] v = c \ beta [/ math] y la definición anterior de “movimiento en el momento adecuado”:

[matemáticas] v ^ 2 + {v_ \ tau} ^ 2 = c ^ 2 [/ matemáticas]

Esto es todo: cada objeto viaja a la velocidad de la luz, pero para los objetos que no viajan a la velocidad de la luz en el espacio, se distribuye sobre el movimiento en el espacio y el “movimiento en el momento adecuado”. No estoy seguro de por qué los libros de texto de física que enseñan relatividad no usan esta forma de ver las cosas, pero creo que transmite mucho más claramente lo que la relatividad realmente nos dice que está sucediendo.
Antes de intentar responder a su pregunta, tenga en cuenta esto: si

[matemáticas] v = 0 [/ matemáticas]

entonces lo anterior se reduce a

[matemáticas] v_ \ tau = c [/ matemáticas]

Esto significa que, ahora mismo, mientras está sentado frente a la computadora, ¡está viajando a la velocidad de la luz! La razón por la que no se da cuenta de esto es que su movimiento se distribuye por completo en “movimiento en el momento adecuado”. Como nunca puede estar en movimiento con respecto a usted mismo, siempre viaja a la velocidad máxima en el momento adecuado en su marco de descanso. Pero cuando algo viaja en el espacio en relación con usted, entonces su movimiento, según lo observado por usted, se “redistribuye” sobre el movimiento en el espacio, por lo que, según lo observado por usted, no viajarán tan rápido en el tiempo adecuado como usted, lo que es decir, observas que “envejecen” más lentamente que tú, y volvemos a la relación anterior entre el tiempo apropiado y el tiempo coordinado.
Permítanme ahora abordar por qué no pueden romper la velocidad de la luz en el espacio. Bueno, para romperlo, primero debes alcanzarlo. Entonces, ¿por qué usted, o cualquier otro observador del espacio-tiempo, no puede alcanzar la velocidad de la luz en el espacio? Si siguió la cadena de razonamiento anterior, ahora está en una excelente posición para reformular la pregunta en una equivalente que pueda acercarlo a una respuesta satisfactoria, a saber: ¿Por qué no puede usted, o cualquier otro observador del espacio-tiempo, viajar a cero? “movimiento en el momento adecuado”?
Considere lo que significa viajar a cero “movimiento en el tiempo apropiado”:
Para objetos que viajan en [matemáticas] v = c [/ matemáticas] o equivalente

[matemáticas] v_ \ tau = 0 [/ matemáticas]

si tales objetos tuvieran un marco de descanso espacio-temporal, entonces en su marco de descanso, el momento en que se observa que comienzan a existir estaría separado por el momento en que desaparezcan en exactamente cero segundos. Esto se deduce directamente de la fórmula estándar

[matemáticas] c \ tau = c \ frac {t} {\ gamma} = t \ sqrt {c ^ 2-c ^ 2} = 0 [/ matemáticas] (ya que v = c aquí)

eso relaciona el tiempo apropiado para coordinar, o simplemente puede conceptualizarlo por el hecho de que para [math] v_ \ tau = 0 [/ math] no se atraviesa ningún intervalo de tiempo apropiado. Esto significa, en particular, que si tales objetos tuvieran un marco de descanso espacio-temporal, ¡observarían que su propia duración de existencia en el espacio-tiempo sería exactamente cero! (Yo llamo a esto la “paradoja de la existencia de fotones” porque ingenuamente, si no supiéramos que los fotones existen, entonces seguramente esto se habría utilizado como argumento para justificar por qué no existen).
Ahora imagine que de alguna manera podría transformarse en un cuadro en el que su movimiento en el momento adecuado es cero. Entonces compartirías tu marco de descanso con objetos que observan su propia duración de existencia en el espacio-tiempo para ser exactamente cero. Pero, ya tenías una duración de existencia distinta de cero en el espacio-tiempo antes de transformarte a ese marco. Como resultado, si fuera posible para usted alcanzar la velocidad de la luz, entonces un observador podría observar en su marco que su propia duración de existencia en el espacio-tiempo sería cero y no cero.
Esto es una contradicción, por lo que significa que no puede transformarse en un cuadro en el que su movimiento en el tiempo adecuado es cero, es decir, usted (o cualquier otro observador del espacio-tiempo) no puede transformarse en un cuadro en el que viaja a la velocidad de luz, y como no puedes alcanzar la velocidad de la luz en el espacio, tampoco puedes romperla, ya que para romperla, primero debes alcanzarla.
Por lo general, la relatividad se construye a partir de dos postulados, uno de los cuales es la suposición de que la velocidad de la luz es la misma en el marco de cada observador del espacio-tiempo (en realidad, el propio Einstein fue más sutil que eso, vea el artículo “Dos mitos sobre la relatividad especial” por Ralph Baierlein). Ese postulado ya es suficiente para obtener una contradicción directa con la suposición de que un observador del espacio-tiempo puede alcanzar la velocidad de la luz, pero transmite poca información física, por lo que puede ser más satisfactorio cambiar la contradicción a otra área donde quizás sea intuitivamente más significativo y eso es lo que intenté hacer.

En aras de la exhaustividad, permítanme abordar también lo que sucedería si de alguna manera pudiera “saltar” el límite [matemáticas] v = c [/ matemáticas] y pasar directamente del movimiento subluminal en el espacio al movimiento superluminal.
Observe si [matemática] v> c [/ matemática] entonces [matemática] {v_ \ tau} ^ 2 <0 [/ matemática] lo que significa que cualquier cantidad de "movimiento en el tiempo apropiado" debe multiplicarse por [matemática] \ sqrt {-1} = i [/ math], la unidad imaginaria.
Matemáticamente, [matemática] i [/ matemática] se entiende bien: puede dibujar un plano, llamado “plano complejo” en el que el eje x corresponde a la línea real y el eje y corresponde a la línea imaginaria (es decir, cualquier cantidad a lo largo de este eje se multiplica por [math] \ sqrt {-1} [/ math].
Observe que las direcciones de los dos ejes en ese plano son perpendiculares. Esto implica que si su “movimiento en el momento adecuado” se multiplica por [math] \ sqrt {-1} = i [/ math], entonces ya no “envejecería” en la misma “dirección” que cuando era todavía se mueve subluminalmente en el siguiente sentido: en su marco de descanso “envejecería”, pero en comparación con los observadores del espacio-tiempo ordinarios, no lo haría.
Entonces, otra forma de caracterizar la razón fundamental por la que no puede ir más allá de [matemáticas] c [/ matemáticas] es que hacerlo es equivalente a “redirigir” la dirección en la que “envejece”. ¡Buena suerte con eso!

(Nota para los conocedores de la relatividad: existe un argumento bien conocido de que viajar más rápido que la luz implica viajar en el tiempo. En mi opinión, este argumento es engañoso porque se basa en la relatividad de la simultaneidad en el espacio real y no en una comparación de paso del tiempo entre observadores del espacio-tiempo ordinarios y aquellos que pueden (hipotéticamente) exceder la velocidad de la luz.
Si se estipula el tipo de situación que implica viajar en [math] v> c [/ math] a partir de la cual se puede construir este argumento y donde se supone que el observador debe encontrarse con su yo anterior, el punto espacio-tiempo del primero y el del posterior los observadores todavía están separados en el espacio complejo (porque el observador posterior, mientras viaja superluminalmente, envejece [matemática] [/ matemática]), y solo se “proyecta” al mismo punto espacio-temporal en el espacio real. Creo que uno no puede simplemente ignorar el espacio complejo en esta situación porque admitir [math] v> c [/ math] es equivalente a extender el espacio real al espacio complejo).

Creo que hay implicaciones más profundas de la relatividad especial que aún no se han explorado y que, una vez que lo estén, nos permitirán encontrar respuestas que sean más directas que las que podemos dar ahora, pero espero que lo anterior se traduzca Las limitaciones al menos un poco más intuitivas.

Hemos confirmado muchos de los fundamentos de la relatividad. Entonces, ya podemos medir el hecho de que se necesitaría una cantidad infinita de energía para que un objeto con masa alcance la velocidad de la luz a través de la aceleración, en base a la aceleración real de los electrones y protones a cerca de la velocidad de la luz y ver la cantidad de fuerza incrementos necesarios de acuerdo con las fórmulas derivadas de la relatividad. Es bastante básico extrapolar desde acelerar un protón hasta cerca de c y acelerar una nave espacial hecha de muchos protones y neutrones hasta cerca de c.

Además, hay consecuencias de que algo pueda viajar más rápido que la luz. Una sería que la materia FTL se escaparía de los agujeros negros (por lo tanto, no serían negros). Podemos observar agujeros negros porque doblan la luz de los objetos a distancia. Pero ninguno de estos agujeros negros está emitiendo nada detectable. Otras consecuencias incluyen la ruptura de la causalidad, etc.

Además, está utilizando los términos de manera muy flexible. La relatividad no dice que “nada” puede viajar más rápido que la luz. La no información puede. Para un tercer observador, podemos presenciar el cierre de dos objetos entre sí en una suma mayor que c. Podría haber partículas hipotéticas (como taquiones) que van más rápido que la luz. Y el espacio-tiempo se puede deformar teóricamente de tal manera que los objetos en dos partes del espacio se muevan a una velocidad relativa uno hacia el otro y que se exceda la velocidad de la luz.

La relatividad solo dice que la luz se mueve (exactamente) a la velocidad de la luz y que los objetos masivos no pueden alcanzar o superar esa velocidad.

Puede encontrar este artículo de Wikipedia informativo: Más rápido que la luz – Wikipedia

Las personas que usan intuiciones euclidianas para enmarcar su comprensión de la realidad física a menudo interpretan la afirmación del físico de que “nada puede moverse a través del espacio más rápido que la velocidad de la luz” [i] para significar que los humanos aún no han encontrado una manera de impulsar algo más allá de esta velocidad . Tienden a responder con la sugerencia de que la tecnología avanzada eventualmente permitirá a la humanidad superar esta restricción, pero han perdido el punto por completo.

Los físicos no están haciendo un reclamo sobre el potencial humano. Lo que estamos diciendo es que dentro del vacío, no existe una velocidad definible que exceda la velocidad de la luz. La opción de ir más rápido que la velocidad de la luz desaparece por completo, porque el significado de la velocidad misma se maximiza a la velocidad de la luz. No hay velocidad definible más allá de ese punto porque la Naturaleza no es geométricamente euclidiana.

Cuando aprendemos a comprender la Naturaleza a través de su verdadera forma geométrica, este hecho no es más fantástico que señalar que no se puede ir más al norte que el Polo Norte, y que no se puede tener un color más rojo que el color exacto definido como rojo. Estas declaraciones son verdaderas por definición. Son tautologías. Por la misma razón, por definición geométrica, la naturaleza posee una velocidad límite. Exploremos esto con mayor detalle.

La velocidad de un objeto a través del espacio es igual a la cantidad de espacio que atraviesa (desde el punto de vista del observador) dividido por la cantidad de tiempo experimentado por el observador durante ese intervalo. Esta definición establece críticamente un límite finito para la velocidad máxima en el espacio.

Para explorar cómo, imaginemos que desplegamos un poderoso cohete, o una máquina milagrosa intergaláctica gigante que posee la capacidad de acelerar constantemente con la fuerza de un g durante un período de 10,000 años. Durante todo el recorrido del cohete, su velocidad aumentará cada segundo en 9.8 metros por segundo (desde el punto de vista de aquellos en el cohete). Debido al empuje constante de los motores del cohete, aquellos a bordo sentirán una aceleración constante y uniforme. A medida que se acelera, aumenta la velocidad del cohete. Como consecuencia, la experiencia del tiempo del cohete comienza a disminuir en relación con la experiencia del tiempo de la Tierra. La importancia de esto es que, aunque todos a bordo del cohete continuarán sintiendo una aceleración constante de un g, los observadores de la Tierra verán que la aceleración del cohete disminuye asintóticamente hacia cero, a medida que la velocidad del cohete aumenta asintóticamente hacia la velocidad de la luz.

Este límite de velocidad asintótico permanece exactamente igual (se aproxima al mismo valor límite) independientemente de la magnitud de la aceleración que elijamos para nuestro cohete. Esto nos dice que la velocidad límite en la Naturaleza tiene algo que ver con la forma en que el tiempo se cambia por espacio a medida que aumenta la velocidad. Como este límite representa el punto en el que los relojes del cohete se han detenido por completo, posee una asociación infinita. Si la nave alcanzara la velocidad de la luz, se movería a través del espacio sin experimentar ningún tiempo. Si la velocidad se definiera como la distancia que recorre un objeto, dividida por la cantidad de tiempo que el objeto experimenta durante ese viaje, entonces la velocidad de la luz nos daría un valor infinito (la medida de distancia distinta de cero dividida por una medida de tiempo cero produce infinito) .

Este valor infinito limitante es una razón por la cual c no es relacional. El infinito es equidistante de todas las ubicaciones. A medida que cambiamos nuestro marco de referencia, cambiamos el valor del numerador en esta ecuación, pero el denominador sigue siendo cero. Un número positivo dividido por cero produce infinito. Esto significa que, en cierto sentido, alcanzar la velocidad de la luz es tocar el infinito.

En cualquier marco de referencia que elijamos, nuestra descripción de la velocidad de un objeto que no experimenta el tiempo debe ser idéntica. Es por eso que c es la única velocidad no relacional. No cambia cuando cambiamos nuestra perspectiva por la misma razón que el infinito permanece idénticamente distante cuando cambiamos de posición.

Si optamos por definir la velocidad como una medida de la distancia que recorre un objeto (en comparación con el observador) dividido por el tiempo experimentado por el objeto durante esa traducción, entonces las velocidades infinitas serían al menos teóricamente alcanzables. Pero, dado que hemos definido específicamente la velocidad de un objeto como la distancia que recorre (en comparación con el observador) dividido por el tiempo experimentado por el observador durante esa traducción, el valor máximo permitido para la velocidad es un valor finito conocido como c de [ii].

Esta respuesta es un extracto modificado de mi libro ‘La intuición de Einstein: Visualizando la naturaleza en once dimensiones’, Capítulo 8.

Para obtener más información sobre este tema, y ​​para descubrir cómo se aclara la teoría de la onda piloto al suponer que el vacío es un superfluido, vea Einstein’s Intuition, disponible en tapa blanda en blanco y negro, tapa blanda a todo color, tapa dura a todo color, un iBook, y como Un audiolibro.

[i] Esta es una condición macroscópica. Para explorar por qué ver Richard P. Feynman, QED: The Strange Theory of Light and Matter, pp. 89-90.

[ii] Para una explicación más técnica de por qué c es una velocidad límite, ver: Mermin (1968, Capítulo 15) y Nahin (1999, pp. 342–353 y Tech Note 7).

Primero, abordemos la pregunta de por qué necesitamos un límite de velocidad. Justo, deja que haya un universo sin límite de velocidad. Esencialmente, proclamar que el límite de velocidad es infinito implica que debe haber algo que viaje a velocidad infinita. Vamos a llamar a esto algo … ¿s-light? (No puedo pensar en ningún otro nombre pegadizo en este momento, así que sigamos con eso)

Debo hacer una pausa aquí para decirte que estamos interesados ​​en el universo en el que vivimos, y no en una loca dimensión paralela. Lo que nos gustaría descubrir es que ¿podemos tener un universo similar al nuestro sin límite de velocidad? Si esto está claro, saltemos directamente.

Si el universo es similar al nuestro, entonces nuestras leyes físicas deberían ser válidas, ¿no? Con algo como la luz s, la energía asociada a ella debe ser infinita, y si la buena luz s antigua tiene energía infinita, el universo debe tener energía infinita; y con esa afirmación, la conservación de energía se va por la ventana (recuerde, sumar o restar cualquier cosa del infinito es el infinito mismo). Sin conservación de energía, la física ya está empezando a parecer desordenada. La termodinámica se ha ido para siempre, y con ella se desmorona la química tal como la conocemos. Para evitar que este sea un escenario Everyone Dies ™, tenemos que ceder y aceptar la existencia de un límite de velocidad.

Suponiendo que el límite de velocidad sea un número c , deje que alguien dispare la partícula que se mueve más rápido (o agite, lo que sea que flote su bote) hacia mí, que viaja hacia la fuente en su batmóvil (lo leyó bien) con una velocidad v . Vería que la partícula / onda se mueve hacia mí con una velocidad que equivale a c + v. Lo que en esencia violará nuestra afirmación de que el límite de velocidad es c . Por lo tanto, no puedo verlo acercarse con una velocidad de c + v . Por lo tanto, cualquier cosa que viaje con la velocidad límite debe tener la misma velocidad en todos los marcos de referencia inerciales. Por lo tanto, llegamos a una conclusión lógica del axioma de Einstein, la velocidad de la luz es de hecho constante en todos los marcos de referencia inerciales.

La última declaración trae consigo grandes consecuencias. Porque si la velocidad de la luz sigue siendo la misma en todos los cuadros, la relatividad galileana se rompe; c + v permanece esencialmente c. A menos que queramos declarar que todas las matemáticas son inconsistentes con la física (que, por cierto, sería una cosa horrible, horrible ), nos propusimos encontrar otras ecuaciones de transformación que se encarguen de este nuevo límite de velocidad encontrado. Como consecuencia de nuestras nuevas ecuaciones de transformación, encontramos que la única forma de permitir que este límite de velocidad fundamental exista es tirar por la ventana la idea de la naturaleza absoluta del tiempo. El tiempo, para nosotros los físicos creyentes, es solo otra dimensión. Según la teoría especial, el tiempo no es especial.

Por último, una respuesta de una línea a la pregunta es, simplemente, que la velocidad de la luz es un aspecto mucho más fundamental de la naturaleza que la sensación del flujo del tiempo. Nuestra intuición tiene un vínculo “especial” con la cuarta dimensión de la naturaleza; La relatividad especial nos enseña a dejar ir.

[1] Parte de la respuesta se toma de una de mis propias respuestas. Ver nota al pie.

Notas al pie

[1] Respuesta de Arpan Akash Ray a ¿Cuál es la razón fundamental de que la velocidad de la luz sea constante en todos los marcos de referencia?

La razón fundamental por la cual la velocidad de la luz no se puede romper en términos simples es que la masa de un objeto a la velocidad de la luz debe ser infinita y para que alcance la velocidad de la luz, se debe proporcionar una cantidad infinita de energía para eso.

Entonces, ¿por qué el universo quiere preservar esta barrera superior? Bueno, la teoría que establece que la velocidad de la luz no puede romperse, la teoría especial de la relatividad fue motivada por la teoría del electromagnetismo que afirmó que el movimiento absoluto es realmente posible. La ecuación que demostró esta característica del electromagnetismo consiste en dos constantes universales, la permeabilidad del vacío y la permitividad del vacío.

Entonces, como son las constantes universales, nunca cambian y siempre tendrán un valor fijo. Los valores de estas constantes fueron decididos por las leyes del universo mismo, en resumen, el universo mismo, nadie sabe por qué estas constantes tienen el valor que poseen, pero afortunadamente estos son valores que tienen y así es como funciona el universo para nosotros. Así es como el universo está sintonizado para nosotros y las leyes del universo para mantener la velocidad de la luz y evolucionar el espacio y el tiempo en consecuencia.

Como han dicho otros, no estamos seguros. Hay ciertas cosas que parecen sugerir que nada puede moverse más rápido que un límite superior en un límite de velocidad universal.

La velocidad de la luz se usa como una constante de proporcionalidad en varias ecuaciones que funcionan para hacer predicciones a partir de nuestras observaciones. Las ecuaciones en electromagnetismo, relatividad, óptica, etc., todas tienen ese valor especial “c” en alguna ecuación y actúa como una constante de proporcionalidad que describe la relación entre ciertas cantidades, como la ecuación para la intensidad de una onda electromagnética y la densidad de energía. de la onda que es [matemáticas] I = cu [/ matemáticas] donde u es la densidad de energía promedio yc es la velocidad de la luz e I es la intensidad. En cualquier onda electromagnética, el cociente E / B (donde E es el campo eléctrico y B es el campo magnético) es la velocidad de la luz.

Hay muchos otros ejemplos, por lo que cambiar la velocidad de la luz anularía muchas de nuestras predicciones y no modelaría adecuadamente el mundo real. Maxwell también utilizó sus ecuaciones que describen el comportamiento de los campos magnéticos y eléctricos para predecir la velocidad de la luz antes de que se midiera y su predicción fuera bastante acertada.

Otra razón para creer que hay un límite superior en la velocidad de la luz se debe a la inercia. A medida que los objetos ganan más velocidad, su inercia aumenta, por lo que se hace más difícil acelerarlos a medida que se mueven más rápido. También ganarán “masa inercial” a velocidades muy altas, por lo que, eventualmente, la energía requerida para acelerar un objeto finalmente alcanza un punto asintótico (donde la energía requerida es infinita). Este límite es la velocidad de la luz.

Otra consecuencia de FTL es una violación de la causalidad que predice la relatividad. Una vez que superas la velocidad de la luz, el tiempo retrocede en cuadros de referencia inerciales. Este es un problema porque viola la segunda ley de la termodinámica y si sucediera en algún momento, veríamos algunos fenómenos muy extraños. Una partícula hipotética conocida como taquión puede viajar más rápido que la luz. Nunca se ha encontrado, pero un atributo de dicha partícula es que puede llegar a su destino antes de comenzar su viaje. FTL no solo viola la causalidad, sino que también viola la conservación de energía.

Entonces como puedes ver. Hay una gran cantidad de razones por las cuales la velocidad de la luz es un límite de velocidad universal que no tiene signos de cambio en el pasado o en el futuro.

La razón fundamental es que la velocidad de la luz es una constante en todos los marcos de referencia inerciales. Este axioma de la relatividad especial ha sido confirmado por experimentos y predicho por la teoría del electromagnetismo de Maxwell.

La velocidad de la luz no se comporta de manera aditiva como una velocidad normal.

Considere nuestra noción de velocidad. Un automóvil que circula por una carretera puede ir a una velocidad de, digamos, 100 km / h. Si hay un tren que viaja paralelamente al automóvil a 90 km / h, el automóvil solo parece que los pasajeros del tren viajan a 10 km / h. Eso demuestra que las velocidades observadas son relativas a la velocidad del observador. Esta es la propiedad aditiva de las velocidades, conocida como relatividad galileana.

Pero la luz no se comporta así. Si está en un automóvil, tren, avión o incluso un cohete que viaja al próximo sistema estelar, todos esos observadores medirían la misma velocidad de la luz, [matemáticas] c [/ matemáticas].

Eso puede sonar extraño, pero eso es lo que observamos.

¿Cuáles son las consecuencias de esta velocidad de luz extrañamente invariable?

En primer lugar, la luz debe ser en cierto sentido fundamentalmente diferente de todo lo demás, porque todo lo demás parece obedecer la propiedad aditiva de las velocidades.

Considere dos observadores en movimiento relativo con una velocidad relativa cercana a la velocidad de la luz. Si ambos observadores miden la misma velocidad de la luz, pero ellos mismos viajan a velocidades muy diferentes, ¿qué debe cambiar para que esto tenga sentido?

¡La respuesta es que las medidas mismas deben cambiar!

La velocidad se compone de una medida de la distancia recorrida dividida por el tiempo empleado. De hecho, debido a que la medición de la velocidad es una relación de las dos mediciones separadas, para que la velocidad sea la misma para ambos observadores, la distancia y el tiempo deben ser diferentes por algún factor común.

Además, si todos los observadores miden la misma velocidad de la luz, [matemática] c [/ matemática], debido a las distorsiones de sus mediciones de longitud y tiempo, puede extrapolar esto a velocidades relativas más rápidas entre observadores y ver que los factores deben tender al infinito como la velocidad relativa tiende hacia, [matemática] c [/ matemática]. Estos son los efectos llamados contracción de la longitud y dilatación del tiempo.

Para el observador en reposo mirando al observador en movimiento, verían que los relojes del observador en movimiento avanzan más lentamente y el observador en movimiento se vuelve más plano. ¡Finalmente a la velocidad de la luz, el tiempo se detiene! Si ese observador en movimiento está en un cohete, el hecho de que el tiempo parece detenerse significa que simplemente no pueden ir más rápido. El empuje requiere una tasa de cambio de impulso, pero el tiempo se ha detenido, por lo que no hay tasas de cambio.

Eso sí, esto es lo que ve el observador estacionario. Para el observador en movimiento es lo de siempre, pero si miraran al otro observador, verían que el otro observador se volvía más plano y sus relojes se desaceleran. Por lo tanto, nadie puede observar algo moviéndose más rápido que la velocidad de la luz.

Entonces surge la pregunta; ¿Qué hace que la luz sea tan diferente que su velocidad sea constante? La respuesta es que no tiene masa. De hecho, todas las partículas sin masa deben moverse a la velocidad de la luz.

Debido a que las partículas sin masa siempre se mueven a la velocidad de la luz, no pueden tener un marco de descanso. Esa es la diferencia fundamental entre partículas sin masa y partículas con masa.

Esto puede sonar loco y contra intuitivo, pero la prueba está en el budín. Casi todo el edificio de la física moderna se basa en la propiedad de que la velocidad de la luz es una constante fundamental. Es tan sólido que la unidad de longitud ahora se define en términos de la velocidad de la luz. Ningún experimento ha encontrado ninguna evidencia que contradiga esta propiedad. Entonces, realmente depende de aquellos que simplemente están descubriendo y preguntándose acerca de este extraño comportamiento tomarse el tiempo para tratar de entenderlo y sus ramificaciones.

Finalmente, me gustaría señalar que toda la discusión ha sido sobre mediciones y observación. ¿Es la dilatación del tiempo un efecto real? Si. ¿Es la contracción de la longitud un efecto real? Si. Sin embargo, puede hacer una contabilidad loca y ver que la velocidad de la luz realmente no limita nada. Por ejemplo, si combina medidas: suponga que sabe que la distancia a la estrella más cercana es de 4 años luz, pero mide el tiempo de viaje usando el reloj de la nave espacial del viajero y es más lento, de modo que solo lee 1 año cuando llegan a la estrella . Entonces podría decir que viajaron a 4 veces la velocidad de la luz (porque ha mezclado las medidas). Del mismo modo, el viajero puede hacer una contabilidad igualmente loca usando la contracción de longitud y llegar a la misma conclusión. Si viaja muy cerca de la velocidad de la luz, toda la galaxia podría aplanarse al ancho de un panqueque, lo que facilita un viaje muy rápido entre dos puntos (para el viajero). Desafortunadamente, aquellos que ha dejado atrás nunca lo volverán a ver en su vida, porque para ellos la galaxia tiene 300000 años luz de diámetro y el viajero nunca excede la velocidad de la luz.

Hoy, el análisis lógico no solo es nítido sino también constante debido a la verificación experimental de cada concepto. La naturaleza receptora de las personas es bastante estable debido al análisis científico. La gente no es rápida en aplicar la tautología teórica para llegar a conclusiones apresuradas. En vista de la estabilidad de la fase de inteligencia, la proyección de la verdadera naturaleza inimaginable de Dios no enfrentará el peligro de la inexistencia. El científico puede no creer ningún milagro y puede decir que nada es inimaginable. Pero el científico debe aceptar el límite inimaginable del universo.

El científico puede argumentar así: – “Cuando el universo es imaginable hecho de energía cósmica imaginable como la causa fundamental, ¿cómo puedes decir que el límite del universo es inimaginable? Al igual que el océano es agua, el límite del océano también debe ser la misma agua. Por lo tanto, el límite del universo imaginable también debe ser imaginable ”. Este tipo de argumento no es aceptable si analiza el punto sutil del límite. Cuando llegas al borde del océano y te paras en él, debes encontrar agua en un lado y la tierra, que no es agua en el otro lado. El conocimiento tanto del agua como de la tierra es necesario para fijar el límite del océano. La tierra no es agua. Si la tierra también es agua, entonces no se alcanza el límite del océano. De manera similar, cuando alcanzas el límite del universo imaginable, debes percibir la naturaleza imaginable del universo por un lado y la naturaleza inimaginable por el otro lado. Si la naturaleza inimaginable también es imaginable, entonces no se alcanza el límite de la naturaleza imaginable.

Cuando su inteligencia no puede imaginar la naturaleza inimaginable, significa que nunca ha alcanzado el límite del universo imaginable. A menos que percibas las naturalezas imaginables e inimaginables, el límite de la naturaleza imaginable no se logra. Por lo tanto, el límite del universo siempre es inimaginable desde el otro lado. Algunos científicos dicen que el diámetro del universo es de 200 mil millones de años luz.

Otro científico se rió de esto al preguntar que si viaja todo este diámetro y alcanza la pared compuesta del universo, ¿qué hay más allá de esa pared compuesta? ¡Un científico dice que el universo está en constante expansión! Este es nuevamente un concepto ridículo, ya que finalmente significa que nunca se puede alcanzar el límite del universo. Por lo tanto, por supuesto, el científico tiene que aceptar la existencia de la entidad inimaginable, que es el otro lado de la frontera del universo. No es el límite si no se logra el otro lado. El científico tiene inteligencia estable y constante para darse cuenta de la verdad en este argumento, a diferencia de un antiguo tautólogo teórico. El elemento inimaginable, que está más allá de los límites de este universo imaginable, se llama Dios.

Cuando este universo es proyectado por Dios, tocarás a Dios al llegar al borde del universo. Nunca puedes tocar a Dios ya que Él es inimaginable. Significa que nunca puedes alcanzar el límite de este universo. Hoy este concepto fundamental se revela debido a la confianza en la facultad analítica estable y estable de los científicos reales. Por supuesto, los científicos conservadores tontos también existen hoy en día en pequeño número y esto es inevitable en cualquier momento. Además, hoy hay mucha demanda para la revelación de este concepto básico en vista del terrorismo violento que surgió de las diferencias entre las religiones.

Incluso la teoría científica como la expansión constante del universo nos dice que el espacio no es inherentemente infinito, sino que se vuelve relativamente infinito a medida que su mente viaja a través de él para tocar su límite.

Esto significa que el espacio no es realmente infinito por sí mismo, sino relativamente infinito y se expande continuamente antes de que su mente alcance su límite. La conclusión es que el límite del espacio nunca puede ser tocado por su mente ya que el espacio se expande continuamente antes de que su mente llegue al límite. La expansión constante del espacio indica el punto principal de que la frontera del espacio, que es el Dios inimaginable, nunca puede ser alcanzada por su mente ( Apraapya Manasaa saha – Veda, Naantosti – Gita). Este es el punto absoluto y el punto relativo es que el espacio se expande constantemente [por] su mente. Debes establecer el conocimiento espiritual sobre este fundamento básico, que es que el Dios absoluto es inimaginable. Todos los demás conceptos deben construirse como castillos sobre esta base sólida.

Tomemos el caso de una corriente de humo proveniente del fuego. Si viaja en el humo, después de un tiempo, puede tocar el fuego. La corriente del humo es finita porque al llegar al límite del humo, puedes tocar el fuego imaginable. Por el contrario, no puedes alcanzar el límite del humo (espacio) si el fuego (Dios) es inimaginable. Por lo tanto, lo finito o infinito del espacio depende de la naturaleza imaginable o inimaginable de Dios, respectivamente . El infinito del espacio no es su característica, sino que se debe a la naturaleza inimaginable de su generador, el Dios.

La constante expansión del universo también habla lo mismo. A medida que viajas a lo largo del universo, se expande para que no puedas alcanzar su límite y tocar al Dios inimaginable. Por esto, la expansión constante nuevamente está relativamente justificada, es decir, con respecto al Dios inimaginable. La expansión constante no es el fenómeno absoluto del universo, pero es un fenómeno relativo ya que el objetivo de la expansión es solo ver que no se debe tocar el límite del universo o de Dios .

El espacio no puede existir en Dios ya que el producto (espacio) no puede existir en su causa (Dios) antes de su generación. Si el espacio existe en Dios incluso antes de su generación, significa que la generación del espacio deja de tener sentido. Se dice que cualquier cosa se genera, si está ausente antes de su generación. Esto significa que el espacio y, por lo tanto, las dimensiones espaciales no existen en Dios y, por lo tanto, nunca se pueden imaginar. La naturaleza inimaginable está, por lo tanto, justificada.

El Dios inimaginable se prueba a sí mismo al realizar eventos inimaginables llamados milagros. No puedes descartar un verdadero milagro como magia. Incluso si descartas los milagros, el espacio infinito con límites inimaginables es la prueba sólida del concepto de existencia de naturaleza inimaginable que indica la existencia de un Dios inimaginable.

La curvatura del espacio a lo largo del límite del objeto prueba que el espacio es algo (energía sutil) y no nada. Por lo tanto, la generación de espacio se convierte en un concepto lógico ya que algo solo se puede generar y no se puede generar nada.

1) No puedes traer finito e infinito al dominio inimaginable (Dios). La razón es que no puede distinguir entre dos artículos inimaginables, ya que cualquier cantidad de artículos inimaginables se convierte en un solo artículo inimaginable. Como puede distinguir infinito (universo) de finito (un árbol), estos elementos infinitos y finitos permanecen solo en el dominio imaginable (creación).

2) En el dominio imaginable (creación), no puede haber absolutamente infinito porque absolutamente infinito significa infinito a todas las referencias (tanto a Dios Krishna como a cualquier ser humano como Arjuna). Este universo es infinito para cualquier ser humano (como Arjuna), pero finito para Dios (Krishna) ya que Dios Krishna dijo que Dios está alrededor del universo ( Sarvamaavrutya tishthati … ). Por lo tanto, este universo es finito para Dios Krishna y, al mismo tiempo, infinito para Arjuna, por lo tanto, es relativamente infinito. Esto significa que el infinito en el dominio imaginable siempre significa relativamente infinito solamente y nunca absolutamente infinito. Cuando Dios Krishna dijo que este universo es infinito, ( Nantostimama … ) significa infinito desde el ángulo de Arjuna en las palabras de Dios. Si tomas el ángulo de Dios aquí también, ambos versos se contradicen entre sí, ya que en un lugar se dice que Dios está alrededor del universo y en otro lugar se dice que el universo es infinito para el ángulo de Dios también. Por lo tanto, el primer verso es desde el ángulo de Dios y el segundo es desde el ángulo de Arjuna.

En este punto, si realiza un análisis más profundo, puede comprender fácilmente que relativamente infinito significa no solo finito para Dios sino también finito para el supuesto teórico del ser humano y simultáneamente infinito para el cálculo experimental del ser humano. Significa que puede asumir que el diámetro del universo tiene cierto valor real fijo, que se representa como X, pero experimentalmente indeterminable para usted. Esto es exactamente lo que dice el principio de incertidumbre en la ciencia. La determinación simultánea de la velocidad y la posición del electrón es experimentalmente imposible y esto no significa que ambos valores no existan realmente. Dado que ambos valores realmente existen, puede representarlos por X e Y. X e Y solo significan que ambos son valores experimentalmente indeterminables, pero valores realmente existentes, que pueden ser calculados por Dios o representables como X e Y en el supuesto teórico de los humanos. seres Del mismo modo, el diámetro del universo es realmente existente (representable en suposiciones teóricas por alguna letra como X o Y o Z) y realmente calculable por Dios, aunque es experimentalmente incalculable por los seres humanos. En tal caso, los siguientes dos pasos son verdaderos y realmente existentes, aunque no podamos calcularlos experimentalmente. Los dos pasos son: – i) diámetro incalculable de universo relativamente infinito + alguna longitud calculable siempre es mayor que el diámetro incalculable de universo relativamente infinito. ii) Diámetro incalculable de universo relativamente infinito: alguna longitud calculable siempre es menor que el diámetro incalculable de universo relativamente infinito. Excepto el valor de ‘alguna longitud calculable’ (conocida por nosotros), ningún ser humano puede calcular valores para los otros elementos en los dos pasos anteriores. Esto no significa que los valores de otros elementos no existan realmente y solo significa que realmente existen, pero que son incalculables experimentalmente para los seres humanos. La ciencia se divide en dos fases: teórica (física) y experimental (física). Los valores no calculables anteriores existen como no calculables solo en la parte experimental. En el supuesto teórico, estos valores existen (ya calculados por Dios) como desconocidos para nosotros.

3) Los valores experimentalmente incalculables (y realmente existentes en el supuesto teórico) de los elementos mencionados en el párrafo 2 anterior, ni siquiera pueden existir realmente en el supuesto teórico, si reemplaza la palabra ‘ infinito ‘ por la palabra ‘ inimaginable ‘ ( de hecho, la palabra infinito no puede existir en el dominio inimaginable como se dice en el párrafo 1). Los dos pasos anteriores en el dominio inimaginable serán de la siguiente manera. i) Inimaginable X + inimaginable Y = inimaginable X o Y. ii) Inimaginable X – inimaginable Y = inimaginable X o Y. Los valores de todos los artículos inimaginables son incalculables tanto por experimento como por supuesto teórico de su existencia tampoco es posible ya que cualquier número de resultados inimaginables sumados, restados, divididos y multiplicados en un solo inimaginable.

4) Por lo tanto, el verso védico dice que algo (X) – mismo algo (X) = mismo algo (X). Este algo (X) significa la palabra ‘ Purnam ‘. Esto es posible solo si X es inimaginable. En el dominio imaginable, donde solo existe el infinito (solo como relativo infinito), incluso si tomas el infinito (en realidad finito para Dios, así como para la suposición teórica de los seres humanos) representado por X, solo da el siguiente paso: – XX = 0. La adición también es similar. En el caso del infinito relativo (tenga en cuenta que el infinito absoluto nunca existe) del dominio imaginable X + X = 2X. La misma adición en un dominio inimaginable es X + X = X. de manera similar, división y multiplicación. Por lo tanto, no se debe confundir que infinito es inimaginable (porque infinito distinguible de finito nunca puede existir en un dominio inimaginable) y aplicar la suma, resta, inimaginable, etc., (X + X = X o XX = X) a infinito (que siempre es relativo existente solo en el dominio imaginable).

5) No debe aplicar estos pasos de dominio inimaginable (X + X = X y XX = X) al infinito del dominio imaginable (no absoluto infinito, pero solo relativamente infinito) que confunde con la ley de conservación de la energía, lo que significa que la parte de energía (la otra parte es materia) del universo permanece constante si alguna energía se convierte en algo y la misma cantidad de materia se convierte nuevamente en una cantidad equivalente de energía desaparecida. Aquí también, la energía, una parte del universo relativamente infinito, puede ser experimentalmente indeterminable infinito, pero existe con un valor real en suposición teórica. La energía, al ser parte del universo infinito, también se vuelve infinita en este dominio imaginable, pero su valor se representa realmente en el supuesto de una letra como en el caso de todo el universo.

El paso en la ley de conservación de la energía es: – X (energía total) -Y (algo de energía convertida en materia) = XY (energía restante). (XY) + Y (misma cantidad de energía resultante de la conversión de materia en energía) = X y, por lo tanto, X siempre es constante. En el dominio inimaginable también, el resultado es constante (inimaginable + inimaginable = inimaginable, que es constante ya que cualquier número de inimaginables resulta como un solo inimaginable). Debido a esta similitud en el resultado, puede confundir que la ley de conservación de la energía también es como un dominio inimaginable y la razón por la que puede pensar es que la parte de la energía es infinita al igual que todo el universo. Esto lleva a la generalización de que infinito e inimaginable son uno y lo mismo. Tal confusión te hace pensar que algo añadido al infinito es constante como en el caso de lo inimaginable. En el caso de la ley de conservación de la energía, el valor de la energía se mantiene constante debido al mecanismo interno de equilibrio entre una cantidad cuántica de energía perdida (al convertirse en algo de materia) y la misma cantidad de energía ganada (la misma ‘algo de materia’ se convierte en energía equivalente). En el caso del universo infinito, algo agregado a él aumenta la cantidad de universo (que es incalculable por experimento, pero existe realmente que se puede representar asumiendo como X).

6) Cuando haces un problema, hay una norma para representar la cantidad desconocida por X. Aquí X significa que la cantidad tiene un valor definido, lo cual es desconocido para ti en este momento. La X representada arriba también es desconocida para ti, pero es desconocida en todos los tiempos. En el problema, usted dice que 1/10 de la porción de X es 10. Este paso determinado por usted lo ayuda a calcular X como 100. A menos que conozca el valor de X como 100 al principio, no puede decir que 1/10 de X es 10. Por lo tanto, el problema trata con las cantidades, que solo son calculables. En el concepto anterior, la X siempre es desconocida para nosotros, que es el diámetro del universo. Cuando se desconoce el diámetro, el centro del universo también es indeterminable y el radio es incalculable. Aunque algunos científicos dicen que el diámetro del universo es de 200 mil millones de años luz, no es aceptable porque nadie alcanza el borde del universo ni siquiera por instrumentos. El viaje del ser humano o incluso su imaginación hasta el límite del universo es imposible (a pesar de que el límite imaginable del universo está terminando en algún momento ya que el universo no es absolutamente infinito sino relativamente infinito, lo que significa que es finito para Dios e infinito para el ser humano, lo que da como resultado que Dios ya calcule el diámetro). El valor del diámetro (del universo finito en realidad) es ciertamente finito, teniendo cierto valor que es incalculable para nosotros y, por lo tanto, se puede decir que el valor del diámetro es X mil millones de años luz. X tiene ciertamente un valor conocido por Dios y nuestra suposición teórica también dice que existe un número desconocido para X. X no es infinito absoluto real (solo significa infinito relativo que existe el valor del diámetro, lo cual es desconocido para nosotros debido a la incapacidad en el cálculo )

Espiritualidad universal para la paz mundial

LO, gracias por la pregunta.

Comencemos con esta creencia fundamental y trabajemos desde allí … no hay necesidad de que yo diga lo que Thad Roberts dice muy bien sobre la creencia / ley de un físico fundamental:

“Los físicos no están haciendo un reclamo sobre el potencial humano. Lo que estamos diciendo es que dentro del vacío, no existe una velocidad definible que exceda la velocidad de la luz. La opción de ir más rápido que la velocidad de la luz desaparece por completo, porque el significado de la velocidad misma se maximiza a la velocidad de la luz. No hay velocidad definible más allá de ese punto porque la naturaleza no es geométricamente euclidiana ”. Fin de la cita.

En pocas palabras, la física no reconoce velocidades más rápidas que la velocidad de la luz en el vacío, porque el hombre no puede ver más rápido que la velocidad de la luz ni quiere definirlo, por lo tanto, más rápido que la velocidad de la luz no es una opción; para ellos.

Y así, dentro de la comunidad profesional de la ciencia del astrocosmos está el postulado de la luz y “¿Cuál es la razón fundamental por la que no se puede romper la velocidad de la luz?” Es su fundamental / regla.

Irónicamente, el universo y su comprensión no siempre están sujetos a tales postulados.

La luz presenta medidas de habilidades únicas, así como debilidades únicas e inherentes.

Einstein escribió sobre la dualidad de la luz: “Parece que a veces debemos usar una teoría y a veces la otra, mientras que a veces podemos usar cualquiera. Nos enfrentamos a un nuevo tipo de dificultad. Tenemos dos imágenes contradictorias de la realidad; por separado ninguno de ellos explica completamente los fenómenos de la luz, pero juntos lo hacen “. [1]

En mi humilde opinión, es más importante actualmente comprender los límites impuestos a la ciencia al creer que postular que la velocidad de la luz es una velocidad máxima, solo porque la ciencia humana no puede “ver” más rápido que la luz. Es importante comprender la física sobre la luz, así como las leyes / fundamentos / medidas limitantes dentro de la ciencia / las matemáticas.

My Quora Blog-Post 1-8 describe mucho sobre las diferencias b / n Gravity & Light, y las conclusiones de las ciencias a partir de ellas. El ToE es la culminación / finalización de lo que es empírico en el universo y las leyes de física / matemáticas, incluida la luz.

Cuando se hacen preguntas sobre la “razón fundamental por la que no se puede romper la velocidad de la luz”, me recuerda una cita favorita de Maxwell

“Ahora mi gran plan, que se concibió en la antigüedad, … es dejar que nada se deje deliberadamente sin examinar. Nada debe ser tierra santa consagrada a la fe estacionaria, ya sea positiva o negativa. Se debe arar toda la tierra en barbecho y se debe seguir un sistema de rotación regular. … Nunca escondas nada, sea hierba o no, ni parezca que deseas ocultarla. … De nuevo, afirmo el derecho de traspaso en cualquier complot de Holy Ground que cualquier hombre haya apartado. … Ahora estoy convencido de que nadie más que un cristiano puede purgar su tierra de estos lugares sagrados. … No digo que ningún cristiano haya encerrado lugares de este tipo. Muchos tienen mucho, y todos tienen algunos. Pero hay extensiones extensas e importantes en el territorio de los Scoffer, los panteístas, los silenciosos, los formalistas, los dogmáticos, los sensualistas y el resto, que son abierta y solemnemente tabúes. … ”

Y entonces, ¿la ‘velocidad de la luz’ se deja deliberadamente sin examinar, tabú como consagrada a la fe estacionaria?

Los documentos, el ToE, el universo dice que no.

douG

Esta pregunta es muy similar a la que respondí hace unos días, aunque la pregunta también incluía una parte de Enredo de Quantum asociada con la comunicación FTL, así que la incluí aquí,
Las otras respuestas en este blog son bastante buenas …

1. Nada puede viajar más rápido que la luz en el medio local alrededor del objeto. El tiempo se detendrá y la duración en la dieta del viaje pasará a cero, en lugar de ir más rápido que la luz. Es la naturaleza recíproca de la causalidad. A veces la velocidad de la luz se llama velocidad de causalidad. El tejido del espacio, la materia, la energía, el tiempo y la masa están atrapados en él.

Sin embargo, al utilizar un “dispositivo de deformación” que puede comprimir y expandir el espacio utilizando ondas de gravedad concéntricas extremas, “puede que” no sea “absolutamente” imposible alcanzar distancias interestelares a velocidades FTL. Sin embargo, la ciencia detrás de la unidad warp solo se imagina matemáticamente, e incluso eso es incompleto, con muchas incógnitas totales. La especulación matemática inteligente con algunos aspectos que bordean la fantasía todavía se aplican.

2. Si bien el entrelazamiento cuántico puede tener dos partículas de tamaño cuántico con los mismos estados entrelazados (u opuestos) (giro, carga, etc.), instantáneamente a grandes distancias, actualmente no es posible (y en el futuro previsible) asociar códigos binarios con estados cuánticos para transmitir información significativa o útil a velocidades FTL.

3. Las ecuaciones de “impulso de urdimbre” del Dr. Alcubierre y el Dr. Nataro (y agregue los tan esperados “cálculos de reducción de energía” del Dr. White) apenas alcanzan unos pocos granos de arena en lo que es una playa muy grande.

Algunos problemas importantes que quedan con la unidad warp son:
– La dependencia de la “energía negativa” para inflar el espacio-tiempo. Si bien esta teoría ha ganado cierta popularidad recientemente en la comunidad científica, todavía hay poca evidencia, si es que hay alguna, de su existencia.

– la capacidad de convertir energía en manipulaciones espacio-temporales. Esto requiere una mayor comprensión de la física de partículas que la que poseemos actualmente, e incluso reivindicar el Bosón de Higgs es solo un paso muy pequeño hacia adelante en ese largo, largo frente.

– Incluso con el Dr. Whites redujo en gran medida los requisitos de energía derivados de las ecuaciones de campo, la energía sigue siendo muy, muy, mucho más allá de lo que podríamos esperar generar, mucho menos CONTROL y MANIPULAR durante muchos cientos de años, si no miles de años . Un blogger (David Beckly) comentó que tomaría la producción de energía del sol de la Tierra para el viaje por deformación. En realidad, las ecuaciones originales del Dr. Alcubierre tienen la producción de energía / masa mucho, mucho, mucho más que solo nuestro sol … pero el Dr. Harold White de la NASA mostró requisitos de energía muy reducidos obtenidos al dar forma al campo de deformación en un parabólico delgado (como el fútbol) suena y pulsa el campo a altas frecuencias … “SI” tiene razón, sus cálculos mostraron el equivalente de energía de masa (reaccionando materia con antimateria) de aproximadamente 1000 KG / seg para una nave Shuttle del tamaño de un SUV a 10 X la velocidad de la luz. Esto es menos de nuestro Sol en 4.5 millones de veces, sin embargo, todavía es más de 10 millones de veces más que la tasa de producción de energía mundial actual … Se necesitaría la energía de explotar 500 T-ZAR Bomba por segundo, por cada segundo de operación en warp para el transbordador de tamaño SUV. T-ZAR Bomba es la bomba de fusión más grande jamás creada con aproximadamente 4.000 veces la bomba de Hiroshima.

– La maquinaria / tecnología real que podría utilizar energía oscura, manejar las enormes energías y comprimir el espacio en sí mismo es actualmente “menos que especulativo” … está tan por delante de nosotros como lo estaría el transbordador espacial para Hypatia en la Biblioteca de Alejandría en 410 DC , al menos. Además, la Ley Moores, el aumento exponencial de la potencia de la computadora NO se aplica a esto. El transporte, las tecnologías de generación de energía y la mecánica pesada y la ciencia de los materiales y la física de alta energía NO avanzan casi tan rápido como las computadoras, los teléfonos inteligentes y similares. Por ejemplo, los automóviles y los jet-liners y cohetes realmente no han aumentado significativamente su velocidad en los últimos 50 años. De hecho, Concord está muerto. … hemos disminuido la velocidad, no es que Concorde haya sido viable.

– Nuclear Fusion para la generación de energía de ganancia positiva, que todavía está más allá de nuestra capacidad, está mucho, mucho, mucho más cerca de la realidad que la velocidad warp. La ingeniería y la ciencia detrás de la fusión del poder están en marcha. La ciencia de la deformación por deformación es principalmente especulativa, con algunas partes que rayan en la fantasía. Fusion for power es otra pequeña tecnología de trampolín, pero es demasiado primitiva y no puede generar las densidades de energía necesarias para deformar el espacio. Tampoco puede la antimateria generar las energías necesarias. Nuestra comprensión de la gravedad es incompleta y ser capaz de generar, manipular, propagar y controlar con precisión la gravedad es un trampolín requerido antes de aprender a usar la gravedad para deformar el espacio.

Por el momento y en el futuro lejano, estos obstáculos son TOTALMENTE PROHIBIDOS, pero “pueden” obtenerse dentro de miles de años.

Mucha gente ha respondido a esta pregunta haciendo alguna variación en los fenómenos observados en la relatividad de la dilatación del tiempo, y el tiempo parece ‘detenerse’ a medida que alcanza la velocidad de la luz.

Sin embargo, esa no es una respuesta muy satisfactoria, básicamente responde a la pregunta diciendo “porque eso es lo que observamos”.

Lo interesante es preguntar * por qué * hay dilatación del tiempo. Y responder diciendo “porque la velocidad de la luz es constante” es realmente solo una tautología.

Una posible respuesta es que el espacio y el tiempo son discretos, no continuos, y que las partículas y las ondas son en realidad efectos que surgen del universo como la operación de un autómata celular reversible, compuesto por muchas ‘células’ pequeñas pero no infinitesimales que tienen un número finito de estados.

En ese caso, uno podría imaginar fácilmente que un fotón podría ser una configuración simple pero ondulada de estados celulares, como un planeador en el Juego de la vida de Conway, que se propaga a velocidad constante (aunque un planeador Conway solo puede moverse en uno de cuatro direcciones, uno puede imaginar reglas ligeramente más complejas que permiten la propagación en una dirección casi arbitraria, pero a velocidad constante), y resulta ser la configuración más simple que puede propagarse, y la más rápida.

Otras configuraciones, que son quizás más complejas, que componen la materia, tienen configuraciones diferentes que hacen que tengan la capacidad de propagarse más lentamente que la velocidad de la luz y acercarse asintóticamente a ella. La contracción de Lorentz se cae naturalmente de este modelo, ya que es más difícil imaginar que una partícula * no * cambie de forma al cambiar su velocidad que imaginar cómo podría mantener su forma y cambiar la velocidad.

A modo de ejemplo, he generado un conjunto de reglas en un autómata celular 3D que permite que un planeador se propague asintóticamente en cualquier dirección y a cualquier velocidad, con un máximo que es esencialmente una “velocidad de la luz” y es un múltiplo simple de ciclo de reloj del sistema. Experimenta una contracción similar a Lorentz en su tamaño en la dirección en que se mueve. Cuanto más tiempo esté en un eje dado, más lento se moverá a lo largo de ese eje.

La regla del “elevador” para un planeador que se mueve a velocidad independiente a lo largo de diferentes ejes

Basado en el trabajo de Fredkin y Miller.

http://52.7.130.124/wp-content/u

Einstein dio específicamente una teoría del principio, no una teoría constructiva (que habría respondido por qué), porque la teoría del átomo no estaba bien desarrollada, y otros factores, que explicó en un artículo del NY Times (28 de noviembre de 1919) . Lo que generalmente escuchamos de los relativistas es la versión basada en principios del argumento, es decir, que si uno acepta la suposición de que la velocidad de la luz es constante para todos los observadores, entonces se deduce que la masa aumenta y el tiempo disminuye y uno no puede ir más rápido, etc. etc. Esto prácticamente invita a las personas a tratar de encontrar soluciones alternativas.

Las explicaciones constructivas ahora son posibles, aunque todavía es difícil dar una perfectamente general que cubra “todo”. Sin embargo, al darse cuenta de que los átomos son en gran medida campos electromagnéticos, con pequeños pedacitos de materia, y que los campos electromagnéticos son los mismos ” cosas “como luz, y la información sobre los cambios en las posiciones de los fragmentos de materia viajará solo a la velocidad de la luz, luego, obviamente, empujará los fragmentos de materia (incluso si fuera posible) más rápido de lo que la luz dejaría atrás los campos y la materia en el mejor de los casos se separaría en sus partículas subatómicas constituyentes.

Si esto no te convence de que tú, personalmente, no puedes ir más rápido que la luz, o incluso a la velocidad de la luz, entonces te sugiero que intentes dispararte fuera de un canon de circo realmente grande. ¿Quizás no te desmorones? Si eso funciona, sigue creciendo.

En serio, y uso el humor solo para aclarar el punto, no para ser condescendiente (aunque, por supuesto, soy culpable), ya que los electrones en los átomos deben ganar más y más energía para rodear el núcleo mientras el núcleo se desplaza a la luz cercana velocidad, pesarán más del aumento de energía. Todo se moverá más lentamente porque gran parte de su estructura interna de campo electromagnético está ocupada simplemente moviéndose a casi la velocidad de la luz. Por lo tanto, todos los procesos, incluidos los relojes, se ralentizan.

En cuanto a la contracción de la longitud, las partículas de campo (fotones) deben correr hacia adelante y luego retroceder para completar sus rutas de unión de moléculas, de modo que las cosas aparezcan comprimidas en la dirección del movimiento.

Otra pequeña arruga con esta explicación pseudo-constructiva es que es difícil dar convincentemente sin parecer apoyar la noción de un marco de referencia absoluto (Ether), lo cual no hago, al menos no hasta que alguien lo mida. Para la persona que se mueve a la velocidad de la luz, todo se ve muy bien en su propio marco de referencia. Um, pero el resto del universo corriendo a la velocidad de la luz parece inflado y acortado en masa, ¡y el cielo ayuda si nos topamos con algo!

More Interesting

¿De dónde viene la restricción energética que resulta en la velocidad de la luz y la relatividad?

Si nuestro universo fuera uno de los muchos en expansión a la velocidad de la luz cercana, ¿qué pasaría teóricamente si nuestro universo chocara con otro?

Einstein descubrió que la velocidad de la luz es el límite máximo. ¿Por qué esta comprensión le hizo pensar en la gravedad?

¿Los procesos biológicos en el cuerpo de una persona se vuelven más lentos si viajan a una velocidad comparable a la de la luz?

Si los científicos descubrieran materia que puede escapar de un agujero negro, ¿sería más rápido que la luz? ¿Cómo afectaría a las leyes contemporáneas de la física?

Si una nave espacial se acercara a la velocidad de la luz, ¿sería capaz de atravesar un planeta sin sufrir daños?

¿Puede una futura sonda estilo Voyager viajar más rápido que los aproximadamente 15,000 m / seg que hacen ambos Voyagers?

¿Por qué la información viaja a la velocidad de la luz?

¿Cuál es la prueba de que la velocidad de la luz no es relativa?

¿Cómo sería ser una persona en la Tierra por teléfono con una persona en una nave espacial que viaja al 10%, 50% o 99% de la velocidad de la luz?

¿Cómo se relaciona la relatividad con viajar a la velocidad de la luz y el tiempo?

Una nave espacial que viaja a la velocidad de la luz podría destruir un planeta al estrellarse contra él. ¿Por qué el Imperio (SW) no consideró esto?

¿Crees que podremos superar la velocidad de la luz después de entrar en un agujero negro?

¿Por qué la energía de movimiento de la luz no puede almacenarse / aprovecharse de la misma manera que la energía solar y térmica?

Si un observador observa cómo una nave espacial se aproxima de frente a la velocidad de la luz, y la nave espacial enciende sus faros, ¿la luz de los faros llegará al observador antes que la nave espacial?